Daily Drills 30 - Section 30 - Question 1

The conclusion above is properly drawn if which one of the following is assumed?

LadyMae January 18, 2019

Assumptions

I am consistently getting these wrong. Can you explain what makes something necessary or sufficient? My thinking was that if something must be assumed for something else to be true, then it must be necessary to the argument. Could you explain the flaw in my logic?

Reply
Create a free account to read and take part in forum discussions.

Already have an account? log in

Ravi January 18, 2019

@LadyMae,

Great question. A necessary assumption (necessary premise) is a
premise that must be true in order for the argument to stand a chance
at being valid. In other words, if a necessary assumption/premise is
false, then the argument would be wrecked and no longer make sense.

A good way to think about necessary premises is that when we're
looking for a necessary premise, we're looking for an answer choice
that, if false, makes the argument lose.

A sufficient assumption (sufficient premise) is a premise that, if
true, makes the argument valid. Basically, if we add this to the
argument, it makes the argument indestructible and makes it win.
However, it doesn't have to be true in order for the argument to be
valid.

A good way to think about sufficient premises is that when we're
looking for a sufficient premise, we're looking for an answer choice
that, if true, makes the argument win.

The question stem reads, "The conclusion above is properly drawn if
which one of the following is properly assumed?"

It sounds like you are misinterpreting the meaning of this phrase. In
what you wrote, you said that if something must be assumed for
something else to be true, then it must be necessary to the argument.
This is true; however, this is not what this question stem is saying.

The question stem is asking us which of the following, IF we assume
one of the answer choices, makes the conclusion follow. In other
words, which of the following answer choices, if true, makes the
argument win.

Does this help? Let us know if you need any further
clarification—we're here to help!